Investments CFP

अब Quizwiz के साथ अपने होमवर्क और परीक्षाओं को एस करें!

When a company, working with an underwriter, offers the investing public a certain number of shares of its stock at a certain price, the company is making what is known as a

public offering.

The Performance Fund had returns of 19% over the evaluation period and the benchmark portfolio yielded a return of 17% over the same period. Over the evaluation period, the standard deviation of returns from the Fund was 23% and the standard deviation of returns from the benchmark portfolio was 21%. Assuming a risk-free rate of return of 8%, which one of the following is the calculation of the Sharpe index of performance for the Fund over the evaluation period?

(.19 - .08) / .23 = .4783 Return-Risk Free/ dev

Joe purchased 1 share of XO for $80. One year later the stock paid a dividend of $2 and Joe purchased an additional share for $95. Joe sold the stock 1 year later for $110. What is the time-weighted return?

18.5% (-80 cfj, 2cfj, 110cfj, orange irr)

Holly bought a stock at the margin requirement, when the stock was trading at $10. The stock paid quarterly dividends of $.25. Holly held the stock for one year and sold the stock when it was trading at $11. What was Holly's holding period return?

40%. HPR= equity/ repayment The first key to this question is knowing that the margin requirement is 50%, which is established by the Federal Reserve. So, Holly is required to pay $10 × .50 = $5 in cash and borrow the other $5 per share to make the investment. The question does not reference any margin interest, so it's excluded from the calculation. The second key to this problem is that the Purchase Price in the numerator reflects both the equity contribution of $5 per share and the $5 per share that must be repaid to the broker. The Purchase Price in the denominator only needs to reflect the $5 in equity paid. HPR = (SP - PP +/- CF) / PP HPR = ((11 - 10) + (.25 × 4)) / (10 × .5) HPR = 40%

David has $20,000 that is earmarked for a down payment on a house in two years. If David is in the 28% tax bracket, what should he invest the $20,000 in?

A 4% tax free money market mutual fund. The tax equivalent yield for the tax free money market fund is 5.56% .04 ÷ (1 - .28) The taxable equivalent yield is greater than the taxable corporate bond paying 5.4%. The mutual fund and intermediate muni-bond fund are not appropriate given the investor's time horizon.

If the actual risk/return performance of a stock lies above the Security Market Line, the stock is said to have a(n):

Actual return that was greater than the required return. Performance of a stock below the SML is a negative alpha. Again, the Jensen formula can be used for this calculation

A convertible bond has a par value of $1,000, a market value of $1,200, and a conversion price of $20. What is the conversion ratio?

Conversion ratio = $1,000/$20 = 50

Annual interest divided by the current market price of a bond.

Current Yield

Which of the following are true statements about the Capital Asset Pricing Model (CAPM)? The Capital Market Line (CML) by itself does NOT determine the optimal portfolio for an investor. Beta is used as a measure of risk on the Security Market Line (SML). The required return is beta times the market return. As investors substitute risky securities in place of risk-free assets, both risk and return increase.

I, II, and IV only. The required rate of return is determined by adding the risk premium (which is the market rate of return minus the risk free rate times the beta) to the risk free rate of return.

The cumulative feature on a preferred stock is best described in the following:

If dividends are not paid in a given cycle, they cannot be paid to anyone else until they are paid to preferred shareholders. Preferred is non-voting stocks.

Which of the following is not a diversifiable risk?

Interest Rate Risk

You purchase one put contract and pay a $3 premium that allows you to sell the stock at $50. The stock is currently trading at $48. What is the intrinsic value of the situation you're in?

Solution: $2 Intrinsic Value of Put = Strike Price - Stock Price, therefore IV = 50 - 48 = 2

Jason purchased a six-month call on ABC stock with a premium of $2.50, when the stock was trading at $38. The strike price was $40. At what market price does Jason just break-even on this investment? Ignore transaction costs and taxes.

Solution: $42.50 Call Instrinsic Value (break even) = Stock - Strike = 2.50; Stock -40 = 2.50; S = 42.50

Bob and Betty have approached you looking for the right hedge against possible, expected future inflation. You suggest to them that they:

Solution: Invest in precious metals. None of the choices are necessarily stellar, but in contrast to the other choices, "Invest in precious metals" makes far more sense, as metals have generally performed well as inflation hedges over time.

Using the constant growth dividend valuation model, calculate the intrinsic value of a stock that pays a dividend this year of $2.00 and is expected to grow at 6%. The beta for this stock is 1.5, the risk free rate of return is 3% and the market return is 12%.

Use the constant growth dividend model to solve for intrinsic value. The question does not provide the required rate of return, however the capital asset pricing model can be used solve for required rate of return. V = D1/(r -g) V = 2 (1.06)/(.165 - .06) V = 20.19 R = Rf + b(Rm - Rf) R = .03 + 1.5(.12 - .03) watch your order of operations here, PEMDAS: .12 - .03 = .09 x 1.5 = .135 + .03 = .165 R = .165

To obtain the maximum reduction in risk, an investor should combine assets that

have a correlation coefficient of negative one.

The stock listing for a company shows a P/E of 18, a dividend yield of 2.4% and a closing price of $23.76. What is the amount of dividends per share?

Dividend Yield = Dividend/Stock Price .024 = X/23.76 X = .024 × 23.76 = .5702

Commercial Paper with a maturity more than 270 days are not permitted under SEC regulations.

False

Interest rate adjusts to protect investor from inflation.

I Bonds

The type of risk which CANNOT be eliminated through diversification is:

Solution: Systematic risk Unsystematic risk, company specific risk and business risk can all be eliminated through diversification.

Match the investment characteristic(s) listed below which describe(s) a unit investment trust. Passive management of the portfolios. Self-liquidating investments usually holding bonds. Both "A" and "B." Neither "A" nor "B."

Both statements are correct because a UIT typically holds municipal bonds until maturity. UITs can also own equities.

Investment A produced annual rates of return of 4%, 8%, 14% and 6% respectively over the past four years. Investment B produced annual rates of return of 5%, 12%, 8% and 11% respectively over the past four years. Which investment was more risky over the past 4 years?

B. (enter each one with the sigma + key, then orange sigma xy key)

Which of the following statements is (are) correct concerning exchange-traded funds (ETFs)? You can buy and sell ETFs any time during trading hours. ETFs are actively managed. ETFs have high portfolio turnover rates. ETFs rarely distribute any capital gains

I and IV only

Assume Mike's stock that he bought at $40 per share falls to $20. How much equity would he be required to contribute per share, if the maintenance margin is 35% and the initial margin was 60%?

Initial transaction" $40 per share, 60% = $24 paid, and debt of 40% or $16 per share. Actual Equity today: $20 − 16 = 4 (current price minus current debt) Required Equity: $20 × .35 = $7.00 Margin Call= 7 − 4 = 3

Averages and indexes differ from one another in that an index:

Measures the current price behavior of a group of stocks in relation to a base value set at an earlier point in time.

A portfolio has a total return of 10.5%, a beta of 0.72 and a standard deviation of 6.3%. The risk free rate is 3.8%, the market return is 12.4%. Jensen's measure of this portfolio's performance is

A = Actual Return - Er = .105 − .0999 = .005 ER = Rf + B (Rm − Rf) ER = .038 + .72 (.124 − .038) = .0999 Measures the abnormal return over the expected return

Jill places a stop limit order to sell at $27 per share on a stock currently trading at $30 per share. What is the loss, per share, if the price quickly falls to $23 per share?

A stop limit order is a limit order at $27. Since the stock quickly falls to $23, Jill is likely to still own the stock and experienced a loss of $7 per share.

Jill places a stop loss order to sell at $27 per share on a stock currently trading at $30 per share. What is the minimum loss, per share, if the price quickly falls to $23 per share?

A stop loss order will turn to a market order at $27 per share. The minimum loss would be $30 - $27 = $3 per share.

What is the yield-to-maturity of a $1,000, 6% semi-annual coupon bond that matures in 5 years and currently sells for $900?

Solution: 8.50% N = 5 × 2 =10 I = ? 4.25 × 2 = 8.5% PV = <900> PMT = (1,000 × .06) = 60/2 = 30 FV = 1,000

Michael has an investment with the following annual returns for four years: Year 1: 12% Year 2: -5% Year 3: 8% Year 4: 18% What is the arithmetic mean (AM) and what is the geometric mean (GM)?

Solution: AM = 8.25%, GM = 7.91% AM = (.12 -.05 + .08 + .18) / 4 = .0825 = 8.25% GM = (1.12 × .95 × 1.08 × 1.18) ^ (1/4) - 1 × 100 GM = (1.356) ^ (1/4) - 1 × 100 GM = 7.91%

Which of the following would not be an investment holding in a Money Market Mutual Fund? Treasury Bills Bankers' Acceptances Bank Certificates of Deposit Repurchase Agreements

Solution: Bank Certificates of Deposit Money Market Mutual Funds will invest in jumbo CDs (aka Negotiable Certificates of Deposit), but not Bank CDs. Jumbo CDs are typically offered through a brokerage firm and are in amounts of $100,000 or greater. Bank CDs are non-negotiable and are in smaller amounts.

The primary difference between open-end and closed-end investment companies would be:

Solution: Closed-end funds sell only a limited number of shares. Closed-end funds offer a limited number of shares, while open-end funds continually create new shares as new monies are obtained. Closed-end funds offer no price guarantees and do not always sell at net asset value.

When an investor feels that some segments of the markets are efficient and some segments are inefficient, what asset allocation strategy would best fit their beliefs?

Solution: Core-Satellite Allocation Market timing is an attempt to anticipate the movements in Equities, suggesting the markets are inefficient. Strategic, Asset Allocation uses a buy and hold strategy across a broad set of asset classes, suggesting the markets are efficient. Tactical Asset Allocation believes markets are inefficient and they seek to outperform market returns based on short term performance. Core-Satellite seeks to hold a core portfolio in index funds or ETFs with smaller portfolio (satellite) to have an active investment strategy to take advantage of market inefficiencies.

A wheat farmer could reduce risk by buying wheat futures to protect against price decreases.

Solution: False The buyer of a futures contract is taking on the obligation to buy the underlying asset at the expiration of the futures contract at the specified price in the contract.The seller of a futures contract is taking on the obligation to sell the underlying asset at the expiration of the futures contract at the specified price in the contract.Since he is a wheat farmer, he would sell a futures contract to guarantee a sales price of his product later.

The type of risk which measures the extent to which a firm uses debt securities and other forms of debt in its capital structure to finance is known as:

Solution: Financial risk Financial risk has to do with the amount of leveraging or use of borrowed funds a firm utilizes to structure its investment and finance its assets.

Asset selection can be achieved through one of four fundamental means. Which of the following is not one of the four means?

Solution: Fundamental Analysis Choices A, B, and D are 3 of the four fundamental means to achieve asset allocation. The fourth option is technical analysis (not listed as an answer choice). Technical analysis uses the historical pricing and volume data to make asset decisions. Both discount cash flow and relative valuation methods are part of fundamental analysis.

Sylvia has a two assets in her portfolio, asset A and asset B. Asset A has a standard deviation of 40% and asset B has a standard deviation of 20%. 50% of her portfolio is invested in asset A and 50% is invested in asset B. The correlation for asset A and asset B is .90. What is the standard deviation of her portfolio?

Solution: Less than 30%. It's not necessary to use the standard deviation of a two asset portfolio formula to answer this question. Since there's a 50/50 weighting for each asset, simply take a simple average of the standard deviations (.40 + .20) ¸ 2 = .30. Since the correlation is less than 1, the standard deviation for the portfolio will be less than the simple average. If correlation was equal to 1, then the standard deviation would be equal to 30%.

Your client has asked you to assist her in examining possible additions to her bond portfolio. She has expressed a desire to minimize risk at this stage in her planning process, and to assure income beginning at the point of her retirement, and lasting throughout. She has a tentative retirement date in seven years at age 65. She has an eighteen year life expectancy in retirement. Which of the following is an appropriate addition to her current portfolio? 25-year AAA-rated corporate bonds with a seven-year maturity. 20 year AAA-rated municipal bonds with a seven-year duration. 25-year AAA-rated corporate zeroes with a seven-year duration. 20-year US Treasury zeroes with a seven-year maturity. 25-year AAA-rated corporate bonds with a seven-year duration.

Solution: TV only. The client is looking for income to begin in 7 years. Therefore anything maturing in 7 years will not provide that income. Zeroes provide no income. She wants something out 25 years, not 20 years. Thus, option "V" is the only one left.

Joe purchased 1 share of XO for $80. One year later the stock paid a dividend of $2 and Joe purchased an additional share for $95. Joe sold the stock 1 year later for $110. What is the dollar weighted return?

Solution: The correct answer is 17.6. CF0 = <80> CF1 = <93> (2 - 95) CF2 = 220 (110 x 2) IRR = 17.60%

All of the following are true regarding ADRs except: ADRs are bought and sold on U.S. markets. ADRs eliminate exchange rate risk. ADRs are denominated in USD. ADRs represent stock of a foreign company held in a foreign bank branch.

Solution: The correct answer is B. ADRs do not eliminate exchange rate risk because the ADR must be converted from USD, to the foreign currency of stock it represents, for valuation purposes.

Small company stocks are yielding 15.7% while the U.S. Treasury bill has a 4.3% yield and a bank savings account is yielding 3.8%. What is the risk premium on small company stocks?

Solution: The correct answer is B. Yield in excess of risk free rate. Risk Premium = .157 - .043 = .114

On December 31, the Gold Standard Company reported the following information on its financial statements: Total current assets $680,000 Total long-term assets $1,850,000 Total current liabilities $490,000 Total long-term debt $975,000 According to this information, the company's current ratio is approximately

Current Ratio = Current Assets/Current Liabilities = 680,000/490,000 = 1.3878

Bond A has a 6% annual coupon and is due in 2 years. Its value in today's market is $900. Bond B has a 10% annual coupon and is due in 4 years. It is priced to yield 12%. Bond C is a 9% zero-coupon bond priced to yield 11% in 8 years.

N=2 i=?= 11.91 PV =<900> PMT=60 FV=1,000

Which of the following forms of the efficient market hypothesis supports technical analysis?

None of the above. Even the weak form holds that technical analysis is of no particular value.

Shares are purchased and sold at end of day net asset value.

Open end funds

The efficient market hypothesis asserts that stocks follow:

Random walk

An investor with a required rate of return of 12.5% is looking at a stock that pays a $3.75 dividend per share, has a growth rate of 6%, and is selling in the market for $60.00 per share. What would you recommend?

Use the intrinsic value formula and the expected rate of return formula to arrive at the correct solution for this problem. Formula: V=(D1)/(r-g) V = 3.75 (1.06)/(.125-.06) V = 3.975/.065 V = $61.1538; therefore, the stock is undervalued since it is trading at $60. You can further verify by: D1 = 3.75(1.06) = $3.98 r = (D1/P) + g (3.98/60) + .06 = 12.63%

A bond's compound annual return assuming it is redeemed prior to maturity.

Yield to Call

A bond's compound annual return assuming coupons are reinvested at the market rate.

Yield to Maturity

Closed-end funds are

less liquid than open-end funds. Open end are like mutual funds. Closed are like IPO's.

Beta is the slope of the best fit line for the points with coordinates representing the _______ and the _______ for each one of several years.

market rate of return; security's rate of return

Maintenance margin is the

minimum amount of equity that an investor can have to avoid a margin call

Comparing the variability of a portfolio's return to the market indicates: diversification The Combined Return of a portfolio is the: weighted average return The Combined Standard Deviation of a portfolio is the weighted average: adjusted for correlation

no further explanation

Measures total risk: Standard Deviation Measures systematic risk: Beta Measures the degree of a portfolio's diversification: R-squared to the market

no further explanation

The primary objective of an equity-income fund is:

Current income with less capital risk.

This approach to security analysis starts with economic analysis, moves to industry analysis and then fundamental analysis.

Top-down analysis.

Private Activity Revenue bonds are not Federally tax exempt because they benefit private projects.

False

If fund A has a correlation coefficient of .9, then how much of its return is due to systematic (or market) risk?

Solution: The correct answer is .81. r = .9, r2 = .81; therefore, 81% of the return for the fund A is due to the market.

The Sharpe's measure for Jane Smith's investment portfolio is 0.40, while the Sharpe's measure for the market is 0.30. This information suggests that Smith's portfolio

exhibits superior performance because its risk premium per unit of risk is above that of the market. The higher the Sharpe and Teynor ratio the better.

In computing portfolio performance, the Sharpe index uses ______________, while the Treynor index uses ________________ for the risk measure.

standard deviation; beta

The following data has been gathered concerning a particular investment and conditions in the market. Risk-free rate3.0%Market return10%Beta of investment1.5 According to the Capital Asset Pricing Model, the required return for this investment is

Solution: The correct answer is C. The expected return on the investment. ER= Rf + B (Rm - Rf) = .03 + 1.5 (.10 - .03) = 13.5

The primary objective of an equity-income fund is

current income with less capital risk.

Which of the following performance measures is a stand-alone metric?

Solution: Jensen's Alpha Sharpe and Treynor are performance measures that are used in comparison to each other. Standard Deviation is a measure of absolute risk. Jensen's Alpha is an absolute measure of performance and is widely interpreted as a measure of excess returns.

Passively managed, with individual proportionate ownership.

Unit investment trust

Which of the following statements concerning duration are correct? Duration is a weighted-average time until cash flows are received. The Macaulay duration considers the timing of a bond's cash flows. The Macaulay duration uses the YTM of a bond to discount the cash flows. For coupon bonds, duration will be less than the actual time to maturity.

All the above

Of the following investment, which is designed to provide growth and income? Raw land. Fixed premium annuity. Non-participating mortgage real estate investment trust (REIT.) Convertible bond.

Convertible bond Raw land may appreciate, but provides no income. A fixed premium annuity provides income, but no growth. Mortgage REITs offer income but no growth. Convertible bonds offer income as a bond and growth potential when converted to a stock.

Which one of the following types of investor benefits most from the tax advantage of preferred stocks?

Corporate. There is a 70% exclusion from taxation on the dividends of preferred stock of one corporation held by another corporation. Actually, if 20% or more of the corporation paying the dividend is owned by the company receiving the dividends, then up to 100% of the dividend is tax free. Non-profit institutional stocks generally pay no federal taxes and therefore, the corporate dividend received deduction of 70% is of no value.

You purchase one call contract and pay a $3 premium that allows you to buy the stock at $50. The stock is currently trading at $48. What is the intrinsic value of the situation you're in?

Solution: 0 Intrinsic Value of Call = Stock Price - Strike Price, therefore IV = 48 - 50 = -2, however intrinsic value cannot be negative.

Bonds issued by agencies of the US government are explicitly backed by the full faith and credit of the federal government.

False, Direct issues of the US Government are backed by the full faith and credit of the US Government. Agencies of the US Government are not backed by the full faith and credit of the US Government, with the exception of Ginne Mae.

Given the following diversified mutual fund performance data, which fund had the best risk-adjusted performance if the risk-free rate of return is 5.7%? Fund A: Average rate of return = .0782, Standard deviation of annual return = .0760 and Beta = 0.950 Fund B: Average annual return = .1287, Standard deviation of annual return = .1575 and Beta = 1.250 Fund C: Average annual return = .1034, Standard deviation of annual return = .1874 and Beta=0.857 Fund D: Average annual return = .0750, Standard deviation of annual return = .0810 and Beta = 0.300

Fund D because the Treynor ratio is highest. If a fund is diversified, use the Treynor model and the result there is arrived at by dividing the return by the beta. In this case, fund D has the highest risk adjusted rate of return. Treynor = [(rp-rf)/(Bp)]. In this case, the result is .06. Fund D....075-.057/.3= .06

Jennifer is in the 30% federal tax bracket and 5% state income tax bracket. Jennifer purchased a municipal bond, issued in her state, yielding 4.1%. What is her taxable equivalent yield assuming she does not itemize on her tax return?

Solution: 6.3% T.E.Y. = .041 = .041 = .0631 1 - (.30 + .05) 1 - .35

What is the weighted average beta of the following portfolio? Stock L has a beta of 1.45 and constitutes 10% of the portfolio, stock M has a value of $125,000 with a beta of .93, while stock N makes up 40% of the portfolio with a beta of .65, and stock O, with a 2.2 beta has a dollar value of $175,000.

Please be certain to avoid rounding errors in arriving at the correct solution. For weighted average beta, use only the two places to the right of the decimal. Stock L = 10%; Stock M=$125,000; Stock N=40%; Stock O=$175,000; L+N=50%; therefore M+O=50% or $300,000. Thus the total portfolio value is $600,000. Stock L = $60,000/$600,000 = .10 × 1.45 = .14; Stock M = $125,000/$600,000 = .2083 × 0.93 = .19; Stock N = $240,000/600,000 = .40 × 0.65 = .26; Stock O = $175,000/$600,000 = .2917 × 2.2 = .64. Adding the results (.14+.19+.26+.64=1.23) will result in the weighted average beta of 1.23.

Which of the following is NOT a similarity between preferred stock and debt instruments? Preferred stock represents the same level of risk as debt to the buyer. Preferred stock pays a fixed income in its dividend. Preferreds are purchased for their income stream. Preferred stock is subject to interest rate and purchasing power risks.

Preferred stock represents the same level of risk as debt to the buyer. Preferred stocks are riskier than debt due to the lack of a maturity date on preferred issues.

My margin requirements are 50% initial margin and 25% maintenance margin. I purchase a total of 200 shares at $100 per share using full margin amount for the 200 share purchase. Shortly thereafter, share prices fall to $50 per share. What will my margin call be?

Required: 50 × .25 = 12.50 Actual: 50 - 50 = 0 $12.50 per share × 200 = $2,500

A company has 2 million shares of common stock outstanding. Annual sales are $26 million. The net profit margin is 8% and the dividend payout ratio is 40%. Currently the stock trades at $17.68 per share. Given this information, the company has a P/E ratio of

See screen shot: PE ratio

Which risk adjusted performance is best for each description?

Sharpe Only: A sector mutual fund with a r-squared (to the S&P 500) of .59 Sharpe and Treynor: Relative risk adjusted performance measures Treynor and Alpha: A well diversified mutual fund with a r-squared (to the S&P 500) of .88 Alpha Only: Absolute risk adjusted performance measures

Your client owns a DGL Corporation convertible bond that has a coupon rate of 8% paid semi-annually and matures in five years. Comparable debt yields 7% currently. The GGL bond is convertible into 22 shares of common stock. The current market price of the underlying stock is $52. What is the conversion value of this convertible bond?

Solution: $1,144 Use the formula for calculating conversion value of a bond. Keep in mind the conversion value may be different from the intrinsic value of the bond (which in equilibrium is the market price of the bond.) CV = (PAR ÷ Cp) × Ps CV = (1000 ÷ Cp) × $52 CV = 22 × $52 CV = 1,144 Cp = conversion price PAR / Cp = conversion ratio Ps = Price of the stock Recall that CR = 1000 ÷ Cp = 22 shares per the problem

In the absence of a true risk-free rate, which of the following serve as substitution?

Solution: Treasury Bill Rate Prime rate is the rate banks offer their best customers. Treasury Bonds are long term rates and would not reflect short term changes in rates. Money Market rates are a function of the underlying investment options. T-Bill rates serve as the risk-free rate.

The current annual dividend of ABC Corporation is $2.00 per share. Five years ago the dividend was $1.36 per share. The firm expects dividends to grow in the future at the same compound annual rate as they grew during the past five years. The required rate of return on the firm's common stock is 12%. The expected return on the market portfolio is 14%. What is the value of a share of common stock of ABC Corporation using the constant dividend growth model (round to the nearest dollar)? (CFP® Certification Examination - Released 3/95)

Step 1 - find the growth rate PV = -1.36 FV = 2 N = 5 Solve for i Answer is 8.0185 Next use the dividend growth model D1 R - g 2(1 + .08) .12 - .08 Answer is $54

Jim and Anne Taylor are baby boomers who would like to add an equity investment to their portfolio. They require a 12% rate of return and are considering the purchase of one of the following two common stocks: Stock 1: dividends currently are $1.50 annually and are expected to increase 8% annually; market price = $35 Stock 2: dividends currently are $2.25 annually and are expected to increase 7% annually; market price = $50 Using the dividend growth model, determine which stock would be more appropriate for the Taylors' to purchase at this time:

Stock 1: v = (1.50 × 1.08) ÷ (.12 - .08), v = $40.50 vs Market Price of $35, therefore this stock is undervalued. Stock 2: v = (2.25 × 1.07) ÷ (.12 - .07), v = $48.15 vs Market Price of $50, therefore this stock is overpriced.

Bob Conrad's investment portfolio consists of several types of stocks, bonds, and money market instruments. The portfolio has an overall standard deviation of 12%, a beta of 1.06, and a total return for the year of 11%. Bob is considering adding one of two alternative investments to his portfolio. Stock A has a standard deviation of 13%, a beta of .87, and a correlation coefficient with the portfolio of .6. Stock B has a standard deviation of 11%, a beta of .97, and a correlation coefficient of .95.

Stock A, because it has a lower correlation coefficient. In the process of adding new investments to a portfolio, the lowest correlation coefficient makes the best addition. Closest to negative one (-1) is always best.

Bristol-Buyers Company has a market price of $36.00 per share with earnings of $3.00 per share, a beta of 1.1 and a dividend of $1.20, which means a dividend payout ratio of 40%. Earnings for next year are projected to increase by 25%, and the retention ratio is projected to remain at 60%. Using the price/earnings multiplier, to what level might your client expect to see market prices move in a year?

The $36.00 per share price is divided by the $3.00 earnings per share resulting in a price/earnings multiplier of 12. The increase of earnings by 25% results in a projected $3.75 earnings next year. Stock Price = 12 × $3.75 = $45.00

Which method of portfolio evaluation allows the comparison of a portfolio manager's performance to that of the over-all market using just one calculation?

The Jensen Model.

According to fundamental analysis, which phrase best defines the intrinsic value of a share of common stock?

The discounted value of all future dividends. Intrinsic value (what its worth) is the discounted value of a future stream of cash flows. In the case of a stock, its dividends.

According to the expectations hypothesis, investors' expectations of decreasing inflation will result inThe expectations hypothesis explains the shape of the yield curve based on inflation expectations.

The expectations hypothesis explains the shape of the yield curve based on inflation expectations. If inflation is expected to be lower in the future, then the yield curve is sloping downward.

Which one of the following statements best describes a firm commitment?

The investment banker agrees to purchase the entire issue and resell the securities to the public.

Which of the following best describes a long hedge position?

The investor is short the underlying commodity and long the futures contract.

Large blocks of listed stocks can be traded outside the exchange in the ...

Third Market

American depository receipts (ADRs) are for the following purpose(s): Finance foreign exports. Eliminate currency risk. Sell U.S. Securities in overseas markets. Trade foreign securities in U.S. markets.

Trade foreign securities in U.S. markets.

If the yield curve is inverted and you believe inflation will be coming down, you should buy long term bonds.

True

A twenty year zero coupon bond is more volatile in response to interest rate changes than a similar bond paying a 5% coupon.

True, zero coupon bonds are more volatile

Which of the following forms of the efficient market hypothesis supports fundamental analysis?

Weak Form


संबंधित स्टडी सेट्स

Common Classroom Questions & Phrases (Español I)

View Set

Leccion 14.1 and 14.2 SPA 201 Study Exam

View Set

Harr MLS Review Chemistry 5.4 Calculations, Quality Control, and Statistics

View Set

Psych exam 2 Chapter 6 Practice questions

View Set